2x2 Matrix Differentialgleichungssystem lösen

Neue Frage »

Knightfire1234 Auf diesen Beitrag antworten »
2x2 Matrix Differentialgleichungssystem lösen
Meine Frage:
Hallo,

Ich habe die folgende Matrix und den Anfangswert:
[attach]45089[/attach]

Ich muss dieses DGL lösen.

danke im Vorraus.

Meine Ideen:
Ich zunächst die Eigenwerte bestimmen, was hier einfach nicht geht...
Danach muss ich die Eigenvektoren bestimmen und zum Schluss in die allgemeine Formel einsetzen und dann eben y1, y2 (oder halt x(t) wie mans nennt) bekommen...
xb Auf diesen Beitrag antworten »

Das sind komplexe Eigenwerte
Knightfire66 Auf diesen Beitrag antworten »

das heißt? verwirrt

Du bist hier zweimal angemeldet. Der User Knightfire1234 wird daher demnächst gelöscht. Steffen
xb Auf diesen Beitrag antworten »

Bei den Eigenwerten ist noch ein i dabei
Das sind Schwingungen

5-4i ; 5+4i
Knightfire66 Auf diesen Beitrag antworten »

ok keine ahnung wie man auf diese eigenwerte kommt oder wie man daraus die eigenvektoren bekommt aber, wenn du sagst das sind schwingungen, muss ich dann da was mit sin und cos machen?
klarsoweit Auf diesen Beitrag antworten »

Als erstes wäre es wichtig, daß du weißt, wie man die Eigenwerte und die zugehörigen Eigenvektoren bestimmt. Sonst macht der Rest keinen Sinn.
 
 
Knightfire66 Auf diesen Beitrag antworten »

nein du hast mich falsch verstanden ich meine in diesem fall, weil wie ja komplexe eigenwerte haben... ich kann das im normalen fall... wenn du mir mal ne kleine Einleitung geben könntest...
klarsoweit Auf diesen Beitrag antworten »

Das geht mit komplexen Eigenwerten haargenau wie mit reellen Eigenwerten: bestimme den Kern der Matrix (A - lambda * I) . smile
Knightfire66 Auf diesen Beitrag antworten »

ich weiß ja noch garnicht wie ich die komplexen eigenwerte rausbekomme... ich habe det(A-*I) aufgestellt... und versucht das mit der MNF zu lösen... klappt nicht

also zusammengefasst kommt die formel ^2-10+41 raus

alles in die MNF eingesetzt bekomme ich nun

1,2 = (10 +- )/2

und das mit -64 kann man nicht lösen...
klarsoweit Auf diesen Beitrag antworten »

Zitat:
Original von Knightfire66
und das mit -64 kann man nicht lösen...

Und dafür hat man ja die komplexen Zahlen erfunden. Es gibt exakt zwei komplexe Zahlen, die quadriert die -64 ergeben. Augenzwinkern
Knightfire66 Auf diesen Beitrag antworten »

also ich denke 64*i² ist unsere zahl? da i² = -1 also 64*-1= -64 wieder? also ist die lösen 1,2 = (10+- 8i)/2 ???
klarsoweit Auf diesen Beitrag antworten »

Genau. Siehe den 2. Beitrag von xb. Augenzwinkern
Knightfire66 Auf diesen Beitrag antworten »

ne ich packs nicht, wie lauten denn die eigenvektoren?ich komme für1 = 5 +4i auf was komisches wie v1= (0 0)
klarsoweit Auf diesen Beitrag antworten »

Du mußt doch jetzt den Kern der Matrix bestimmen. Was kommst du da nach Durchlauf des Gauß-Verfahrens?
Knightfire66 Auf diesen Beitrag antworten »

naja...

[attach]45094[/attach]

Hammer
Knightfire66 Auf diesen Beitrag antworten »

also hab mal bisschen weiter rumprobiert:

ich komm die ganze zeit auf:

[attach]45095[/attach]
klarsoweit Auf diesen Beitrag antworten »

Da hast du dich irgendwo verrechnet. Du müßtest auf die Matrix kommen.

Ich muß mich jetzt leider ausklinken. geschockt
Knightfire66 Auf diesen Beitrag antworten »

oh das hab ich auch mal zwischendurch rausbekommen gehabt...

ok kein ding wär nett wenn du antwortest wenn du wieder on kommst, ich mach so lange eine andere aufgabe... smile
Knightfire66 Auf diesen Beitrag antworten »

Also ich kann ja nun die erste Zeile als Spaltvektor schreiben. Dann einfach eine senkrechte dazu finden:



durch Zeilen vertauschen und VZW bei einem:

V1 =

ist das so richtig?

wenn ja warum hat mein Tutor das genau anders herum? also
HAL 9000 Auf diesen Beitrag antworten »

Zitat:
Original von Knightfire66
wenn ja warum hat mein Tutor das genau anders herum? also

Prüfe bitte, ob das dort ein Eigenvektor zu Eigenwert war ... oder nicht vielleicht doch einer zu dem anderen Eigenwert . Augenzwinkern
Knightfire66 Auf diesen Beitrag antworten »

Also der eine lautet

und der andere ist
HAL 9000 Auf diesen Beitrag antworten »

Zitat:
Original von Knightfire66
und der andere ist

Der ist ein Vielfaches von

Zitat:
Original von Knightfire66
V1 =
Knightfire66 Auf diesen Beitrag antworten »

Zitat:
Der ist ein Vielfaches von


ok wie mit was erhält man den einen vom anderen?

und wie komm ich auf

und wie gehts jetzt weiter?

PS: kann das hier bitte ein bisschen schneller gehen ich habe nicht ewig zeit für eine einzige Aufgabe Augenzwinkern
HAL 9000 Auf diesen Beitrag antworten »

Einfach mal nachrechnen! Es ist

,

damit sind beides Eigenvektoren zum Eigenwert .


Zitat:
Original von Knightfire66
PS: kann das hier bitte ein bisschen schneller gehen ich habe nicht ewig zeit für eine einzige Aufgabe Augenzwinkern

Du bist ein Witzbold - wer antwortet denn hier erst so spät: Du oder wir?
Knightfire66 Auf diesen Beitrag antworten »

Zitat:
ok wie mit was erhält man den einen vom anderen?


ok alsoo natürlich... (sry hab gerade mit einer anderen aufgabe zutun und hab mich nicht konzentriert)... und wie mache ich jetzt weiter..

alles in die Formel eingesetzt bekomm ich:

[attach]45101[/attach]

aber mit den komplexen Zahlen komm ich hier nicht klar

Edit:
Zitat:
Du bist ein Witzbold - wer antwortet denn hier erst so spät: Du oder wir?

Bin gestresst wegen der Klausur, sry Gott smile

PS: die Emoji sind echt cool...
HAL 9000 Auf diesen Beitrag antworten »

Mit deinen Image-Scans statt der Verwendung von LaTeX bürdest du uns zusätzliche Arbeit auf - da müssen wir den ganzen Mist erst nochmal eintippen, wenn wir was leicht verändern müssen - dafür gibt's das coole Emoji Forum Kloppe für dich.

Dein ist nämlich falsch: Richtig ist

.

Mit dem gegebenen Anfangswert lassen sich die (ebenfalls komplexen) Parameter bestimmen. Ist dies geschehen, lässt sich abschließend auch alles ins Reelle umschreiben.

EDIT: Sorry, Copy+Paste-Fehler beim Anfangswert - korrigiert.
Knightfire66 Auf diesen Beitrag antworten »

Zitat:
Mit deinen Image-Scans statt der Verwendung von LaTeX bürdest du uns zusätzliche Arbeit auf


-Latex Error, obwohl der code richtig ist
HAL 9000 Auf diesen Beitrag antworten »

Die Leerzeichen müssen weg, d.h., \begin{matrix}, \end{matrix} .

Außerdem ist das Schriftbild schöner, wenn du pmatrix statt matrix verwendest.
Knightfire66 Auf diesen Beitrag antworten »

erstmal danke für die latex tipps...

für die Konstanten C habe ich

HAL 9000 Auf diesen Beitrag antworten »

(in Normalform geschrieben) ist richtig. Und es ist , in diesem Fall lässt sich dein Wert nicht umschreiben, er ist also wirklich falsch.
Knightfire66 Auf diesen Beitrag antworten »

wegen C1:

nach alles einsetzen und so umformen , dass c2 entfällt...







ich glaube das ist immernoch falsch, da ich das nicht umformen kann so dass es deine lösung ergibt...
HAL 9000 Auf diesen Beitrag antworten »

Herrje, muss man denn jede kleine Nebenrechnung nochmal aufs Tapet zerren:



ergibt die beiden Bestimmungsgleichungen

.

Das ergibt



mit den genannten Lösungen.
Knightfire66 Auf diesen Beitrag antworten »

ich weiß du willst nicht so viel mit latex machen aber du könntest ab und zu mal n foto oderso von deinem aufschrieb hochladen (oder machst du das alles im kopf?xD),

also gut C1 = 1/2+i/4...

nun dann sind wir ja fertig, wenn ich die konstanten oben einsetze?

Ich dachte nur es gibt n einfacheren weg, weil jemand (aus der whats gruppe, von unserem Studiumjahrgang) etwas von substitution bei der aufgabe sagte... sagt dir das zufällig was? Kann man die Aufgabe auch anders lösen?
HAL 9000 Auf diesen Beitrag antworten »

Zitat:
Original von Knightfire66
ich weiß du willst nicht so viel latex machen aber du könntest ab und zu mal n foto oderso von deinem aufschrieb schicken (oder machst du das alles im kopf?xD)...

Teils im Kopf, teils eingetippt - kein Papier. Augenzwinkern

Es ist übrigens falsch, dass ich "nicht so viel latex machen" will. Im Gegenteil, ich bevorzuge es - ich bin nur zu faul, deine Sachen in LaTeX zu übertragen und möchte deshalb am liebsten, dass ich sie gleich per Copy+Paste übertragen kann. smile

---------------------------------------------------------

Man hätte pragmatisch auch so vorgehen können, und dabei (nahezu) komplett im "reellen" bleiben: Eigenwerte sind , damit besitzt die allgemeine Lösung auf jeden Fall die Gestalt



mit zu bestimmenden Konstanten . Die Anfangswerte eingesetzt ergibt sich unmittelbar , und weiter dann

,

und die Ableitung ist

,

Der Koeffizientenvergleich liefert mit den Lösungen .
Knightfire66 Auf diesen Beitrag antworten »

Vielen Dank das sieht ja viel besser aus smile

noch paar fragen hierzu:

Zitat:
und weiter dann


[attach]45102[/attach]

Was macht man hier genau?

z.B. erste Zeile: Wenn ich links x1 und x2 habe dann rechts noch b und d unten genauso viele... dann habe ich 2 Gleichungen für 4 unbekannte wie soll das gehen?

Zitat:
und die Ableitung ist


ok ganz normal alles in klammer ableiten und e^(5t) ignorieren, sonnst hätte man ja die Kettenregel anwenden müssen...

Zitat:
Der Koeffizientenvergleich liefert 35=25+4b,15=25+4d mit den Lösungen b=5/2,d=-5/2.

Koeffizienten-vergleich kenne ich aus Partialbruchzerlegung... ok ist klar.. aber woher hast du diese beiden gleichungen rausgeschrieben?

ich glaube du hast wieder viele zwischenschritte übersprungen und ich komm nicht mehr mit... (wir sind ja auch gleich fertig halt durch Big Laugh )
Knightfire66 Auf diesen Beitrag antworten »

Zitat:
z.B. erste Zeile: Wenn ich links x1 und x2 habe dann rechts noch b und d unten genauso viele... dann habe ich 2 Gleichungen für 4 unbekannte wie soll das gehen?

sry habs verstanden, wir dürfen ja ableiten LOL Hammer

Zitat:
Koeffizienten-vergleich kenne ich aus Partialbruchzerlegung... ok ist klar.. aber woher hast du diese beiden gleichungen rausgeschrieben?


damit hat sich diese Frage auch geklärt....

Tanzen

Danke sehr du hast dir echt viel Zeit genommen Gott Gott Gott Lehrer
Knightfire66 Auf diesen Beitrag antworten »

Hallo ich bin gerade fleißig am Wiederholen von alten aufgaben...

ich habe ne frage zu dieser letzten Methode mit trog, Funkt.

ich kann zwar alles Nachvollziehen siehe meine letzten 2 Antworten... aber ich habe ein "technisches Problem"

ich komme einfach nicht auf diese Ableitung... du hast das glaub mit wolfram oder ähnliches gemacht, aber schriftlich geht das nicht so leicht...



hier ist was ich dazu habe... ich komme nicht mehr weiter ganz am Ende... mir fehlen paar schritte:

[attach]46491[/attach]

mfg
Neue Frage »
Antworten »



Verwandte Themen

Die Beliebtesten »
Die Größten »
Die Neuesten »